historians label an event as a turning point when it​

Answers

Answer 1

Answer:

F

Step-by-step explanation:

Answer 2

Answer:

Historians label an event as a turning point when it​ marks the beginning of a distant historical period

Step-by-step explanation:


Related Questions

Jamie invests part of $10,000 at 9% annual interest and the rest at 8%. If his annual income from
these investments is $860, how much does he invest at 8%?

Answers

Answer:

$4000

Step-by-step explanation:

Assuming a = money invested at 9%

Assuming b = money invested at 8%

then we can form am equation from the question given

1) a + b = $10000, also

0.09a + 0.08b = $860 -> multiply by 100

2) 9a + 8b = $86000

Next we try to solve both equations simultaneously. We multiply both the left hand side and the right hand side of the first equation by 8. We have

3) 8a + 8b = $80000, now we subtract 3 from 2

9a + 8b = $86000

-8a - 8b = -$80000

a = $6000.

Then, we plug this in any of the first 2 equations, to get b

9 * $6000 + 8b = $86000

$54000 + 8b = $86000

8b = $86000 - $54000

8b = $32000

b = $32000/8

b = $4000.

Therefore, he invested $4000 at 8%


Find the missing exponent

Answers

Answer:

-6

Step-by-step explanation:

When an exponent is outside and inside the parenthesis you need to add them to get rid of the the parenthesis.  The answer is -6 because -6*2 is -12.

Hope this helps. If you have questions put them in the comments.

Answer:-6

Step-by-step explanation:

When an exponent is outside and inside the parenthesis you need to add them to get rid of the the parenthesis. The answer is -6 because -6*2 is -12.

What’s the answer to 1.9m + 14 = 2m?

Answers

Answer:

[tex] \boxed{ \bold{ \huge{ \boxed{ \sf{m = 140}}}}}[/tex]

Step-by-step explanation:

[tex] \sf{1.9m + 14 = 2m}[/tex]

Move 2m to right hand side and change it's sign

Similarly, move 14 to left hand side and change it's sign

[tex] \dashrightarrow{ \sf{1.9m - 2m = - 14}}[/tex]

Collect like terms

[tex] \dashrightarrow{ \sf{ - 0.1m = - 14}}[/tex]

Divide both sides by -0.1

[tex] \dashrightarrow{ \sf{ \frac{ - 0.1m} { - 0.1} = \frac{ - 14}{ - 0.1}}} [/tex]

Calculate

[tex] \dashrightarrow{ \sf{m = 140}}[/tex]

Hope I helped!

Best regards! :D

Which inequality is true?
O A n-3>1
B. 9n> 27
c. 9/3n> 1
O D. N+7<10

Answers

Answer:

c.

Step-by-step explanation:

Select the relationship that does represent a function

Answers

the 4th one is a relationship that is a fuction I think

Answer:

4th image

Step-by-step explanation:

the others cannot be functions since one domain cannot be in two ranges

If (42)P = 414, what is the value of p?
7 8 12 16​

Answers

Answer:

c

Step-by-step explanation:

Anjali spent half of her weekly allowance
at the movies. To earn more money her
parents let her weed the garden for $10.
What is her weekly allowance if she ended
with $19?

Answers

her weekly allowance is $18
The answer is $18. She ended with $19, her parents gave her $10 for the garden work and she had $9 left. The $9 was from the money she originally had and it was what was left over after spending half of her allowance. She had $18 as allowance.

3. Two fair dice are thrown once, find the probability of getting a total score greater than 5

Answers

Answer:

26/36

Step-by-step explanation:

The probability of something = the amount of favourable outcomes/total possible outcomes.

The total number of dice combinations is 36 (6x6).

In this case there are more outcomes more than 5 than less than 5 so for the sake of simplicity we are going to find the number of outcomes less than 5 and then find the inverse.

There are only 10 possible dice combos which add to totals 5 or under:

1,1 1,2 2,1 1,3 3,1 1,4 4,1 2,2 2,3 and 3,2

That means there is a 10/36 probability of getting a total score less than 5 so to get the probability of getting more than 5, we just do 36/36 - 10/36 (because all probabilities add to 1)

Therefore there is a 26/36 probability that the toatals will add to greater than 5.

Alternatively you could also draw up a possibilities table and count the favourable outcomes.

Hope this helped!

Answer:

13/18.

Step-by-step explanation:

The favourable outcomes are (1,5) (1,6) (2,4) (2,5) (2,6) (3,3) (3,4) (3,5) (3,6) (4,2)(4,3) (4,4) 4,5) (4,6) 5,1) (5,2) 5,3) (5,4) (5,5) (5,6 )(6,1) to (6,6).

= total of 26.

As the total of all possible outcomes is 36 the answer is 26/36

= 13/18.

i already know the answer, its increasing by 4 but i dont know how to write the answer.​

Answers

Answer: m+3

Step-by-step explanation:

Since the jump from each number increases by 3, you would write m+3, since m is indicated in the pattern.

John has a 4 litre bottle of water. One mug holds one half of a litre of water. Jane drinks 3 mugs of water, how many mugs would the remaining water fill

Answers

Answer:

the answer is 3.5

Step-by-step explanation:

1. the total is 4

2.each mug = .5 of a litre

3.jane drinks 3 mugs which=1.5

4.she drinks 1.5 litres which is 3.5 of the whole bottle

5.so the remaining will be 3.5 which is what remains after she drank 1.5 litres of the whole bottle

1. the total is 4 2.each mug = .5 of a liter% 3D 3.jane drinks 3 mugs which = 1.5 4.she drinks 1.5 liters which is 3.5 of the whole bottle 5.so the remaining will be 3.5 which is what remains after she drank 1.5 liters of the whole bottle

Help i dont understand this someone help me pls :(

Answers

Answer:

10

Step-by-step explanation:

-4, -3, 1, 6 That's the correct anwer to this problem

Simon used 13 pears and 9 apple to make fruit salad. What was the ratio of the number of apple to the number of fruit in the fruit salad?

Answers

Answer:

13:9

Step-by-step explanation:

The ratio of the number of apples to the number of fruits in the fruit salad is given by the equation A = 9 : 22

What is Proportion?

The proportion formula is used to depict if two ratios or fractions are equal. The proportion formula can be given as a: b::c : d = a/b = c/d where a and d are the extreme terms and b and c are the mean terms.

The proportional equation is given as y ∝ x

And , y = kx where k is the proportionality constant

It demonstrates the equality of the relationship between the expressions printed on the left and right sides.

Given data ,

Let the ratio of number of apples to the number of fruits be A

Now , the proportion will be

The number of pears = 13 pears

The number of apples = 9 apples

Now , the number of fruits = number of pears + number of apples

Substituting the values in the equation , we get

The number of fruits = 13 + 9 = 22

Now , the proportion of apples to fruits A is

Ratio of the number of apples to the number of fruits in the fruit salad A = number of apples : number of fruits

Substituting the values in the equation , we get

Ratio of the number of apples to the number of fruits in the fruit salad

A = 9 / 22

A = 9 : 22

Therefore , the value of A is 9 : 22

Hence , the proportion is 9 : 22

To learn more about proportion click :

https://brainly.com/question/7096655

#SPJ2

Need help thank you!

Answers

Answer:

$216

Step-by-step explanation:

We know that the cost of an outfit for one team member is 10 + 26 + 18 = $54, therefore, the cost of outfits for all 4 team members will be 54 * 4 = $216.

Consider the composite function (g (x) ) = StartRoot StartFraction 1 Over x squared minus 3 EndFraction EndRoot. If g(x) = x2 – 3, what is f(x)?

Answers

Answer:

C ON EDGE 2020

The function f(x) = [tex]\sqrt{1/x}[/tex] if, F[g(x)] = [tex]\sqrt{(1/x^2 - 3)}[/tex], g(x) = x² - 3, so option C is correct.

What is a function?

The function is a relationship between a set of potential outputs and a set of possible inputs, where each input has a single relationship with each output. This means that if an object x is present in the set of inputs (also known as the domain), then a function f will map that object to exactly one object f(x) in the set of potential outputs (called the co-domain).

Given:

F[g(x)] = [tex]\sqrt{(1/x^2 - 3)}[/tex]

g(x) = x² - 3

Put the value of g(x) in all the options and check whether it is equal to F[g(x)]  as shown below,

Put it in option 3,

f{g(x)} = [tex]\sqrt{(1/x^2-3)}[/tex]

Thus, f(x) = [tex]\sqrt{1/x}[/tex]

To know more about function:

https://brainly.com/question/5975436

#SPJ2

Express the following vector in the form v vi+V2j+ v3k 3u -v if u (3,5,-8) and v (2,-5,7)

Answers

Answer:

7i+20j-31k

Step-by-step explanation:

Given the vectors u (3,5,-8) and v (2,-5,7), they can also be written as;

u = 3i+5j-8k

v = 2i-5j+7k

To express the vector 3u - v in vector form, first let us find the vector 3u;

3u = 3(3i+5j-8k)

3u = 3(3i)+3(5j)-3(8k)

3u = 9i+15j-24k

Subtract v from the resulting vector.

Since v =  2i-5j+7k

3u - v = 9i+15j-24k - (2i-5j+7k)

3u-v = 9i+15j-24k -2i+5j-7k

collect the like terms

3u-v = 9i-2i+15j+5j-24k-7k

3u-v = 7i+20j-31k

Hence the vector 3u-v in the form v =  vi+V2j+ v3k is 7i+20j-31k

Given A is between Y and Z and YA= 14x, AZ= 10x, and YZ= 12x + 48, find AZ

Answers

Answer:

96

Step-by-step explanation:

The value of AZ will be 40.

A is a point between Y and Z and YA = 14x , AZ = 10x and YZ = 12x + 48.

We have to calculate the value of AZ.

What is the value of MP , if N is the midpoint of M and P ?

The value of MP will be equal to the summation of NM and NP i.e., MP = NP + NM .

As per the question ;

A is the midpoint of Y and Z.

⇒ AZ + YA = YZ

⇒ 10x + 14x = 12x + 48

24x = 12 x + 48

⇒ 12x = 48

x = 4

So ,

The value of AZ will be ;

AZ = 10 × 4

AZ = 40

Thus , the value of AZ will be 40.

To learn more about line segments click here ;

https://brainly.com/question/17374569

#SPJ2

If a package of gift cards has a total value of $1050 how much does the average set of 2 gifts card price at if there are 15 in a group?

Answers

Answer:

$140

Step-by-step explanation:

If a package of gift cards has a total value of $1050, the total price for 2 gift cards will be 2×$1050 = $2100

If there are 15 in an average set of 2 gift cards, the average cost of the two gift cards will be expressed as;

Total price of 2gift cards/amount of card per group.

Average cost of of 2 gift cards = $2100/15

= $140

Hence the average cost of 2gift cards if there are 15 in a group is $134

While riding her bike to the ocean and back home 6 times, Mandi timed herself at 52min, 54min, 53min, 51min, 53min, and 43min. She had set a goal of averaging 51 minutes for her rides. How many minutes will she need to spend on her seventh ride to attain her goal?

Answers

Answer:

306 min

Step-by-step explanation:

During its first year in business, Alice's restaurant shows a loss of $12,000. During its second year, the restaurant makes a profit of $23,200, and during its third year, the restaurant shows a loss of $1,200. What is the net profit of the restaurant during its first three years in business?

Answers

Answer:

$10,000

Step-by-step explanation:

-12,000+23,200-1,200= 10,000

The profit of the restaurant during it's first 3 years is 9800 dollars.

What is profit and loss ?

When the amount of money we get is more than what we have invested is more we have profit and when the amount of money we get is less than what we have invested we have loss.

Let us assume Alice start with a net profit or or of 0 dollars.

During its first year in business, Alice's restaurant shows a loss of 12,000 dollars.

∴ He is in loss of (0 - 12000) dollars which is

= - 12000 dollars.
During its second year, the restaurant makes a profit of 23200 dollars.

∴ His net profit is

= ( -12000 + 23000 ) dollars.

= 11000 dollars.

During its third year, the restaurant shows a loss of 1200 dollars so his ne profit is

= ( 11000 - 1200 ) dollars

= 9800 dollars.

learn more about profit and loss here :

https://brainly.com/question/13934673

#SPJ2

What are the graphs to these?

Answers

Answer:

The graph is in the image

Step-by-step explanation:

[tex]y = \frac{3}{5} x -3\\\\2x -y =-4\\\\y=-6,\:x=-5\\[/tex]

400 thousands + 600 thousands in standard form​

Answers

Answer: 1 million

Step-by-step explanation:

Answer:

1,000,000

Step-by-step explanation:

Step 1:

400,000 + 600,000 = 1,000,000

Answer:

1,000,000

Hope This Helps :)

Let A and B be symmetric nxn matrices. For each of the following, determine whether the given matrix must be symmetric or could be nonsymmetric.a) C = A + Bb) D = A^2c) E = ABd) F = ABAe) G = AB + BAf) H = AB-BA

Answers

The matrices C, D, F, and G are symmetric while matrix H is skew-symmetric and matrix E is non-symmetric.

A matrix is an array of elements(numbers) in a peculiar order. A matrix with m rows and n columns is said to be of order m × n.

As it is known that [tex](A + B)^t = A^t + B^t[/tex] and [tex](AB)^t = B^t A^t[/tex] for any n × n matrices A and B.

Also, if matrices A and B are symmetric, then

[tex]A^t = A[/tex] and [tex]B^t = B[/tex].

a) Given that

C = A + B

Take the transpose of matrix C,

[tex]C^t = (A + B)^t[/tex]

    [tex]= A^t + B^t \\= A + B = C[/tex].

Hence, matrix C is symmetric.

b) Given that

D = A²

Take the transpose of matrix D,

[tex]D = (A^2)^t[/tex]

   [tex]= (AA)^t \\= A^t A^t \\= AA \\= A^2 = D[/tex].

Hence, matrix D is symmetric.

c) Given that

E = AB

Let's take an example,

A = [tex]\left[\begin{array}{cc}1&2&2&1\end{array}\right][/tex] and

B = [tex]\left[\begin{array}{cc}1 &-1 &-1&2\end{array}\right][/tex]

So,

E = AB

  [tex]=\left[\begin{array}{cc}1&2&2&1\end{array}\right] \times \left[\begin{array}{cc}1 &-1 &-1&2\end{array}\right]\\= \left[\begin{array}{cc}-1&3&1&0\end{array}\right][/tex]

Clearly, matrix E is non-symmetric.

d) Given that

F = ABA

Take the transpose of matrix F,

[tex]F^t = (ABA)^t[/tex]

    [tex]= A^t B^t A^t\\ = ABA = F[/tex].

Hence, matrix F is symmetric.

e) Given that

G = AB + BA

Take the transpose of matrix G,

[tex]G^t = (AB + BA)^t[/tex]

    [tex]= (AB)^t + (BA)^t \\= B^t A^t + A^t B^t \\= BA + AB = G[/tex].

Hence, matrix G is symmetric.

f) Given that

H = AB-BA

Take the transpose of matrix H,

[tex]H^t = (AB - BA)^t[/tex]

     [tex]= (AB)^t - (BA)^t \\= B^t A^t - A^t B^t \\= BA - AB = -H[/tex].

Hence, matrix H is skew-symmetric.

Thus, the matrices C, D, F, and G are symmetric while the rest are not.

Learn more about Matrix here:

https://brainly.com/question/28180105

#SPJ12

Please Help!! Angles A and B are complementary. If the measure of angle A=3x-8 and measure of Angle B =5x+10, what is the measure is each angle?

Answers

Answer:

Step-by-step explanation:

3x - 8 + 5x + 10 = 90

8x + 2 = 90

8x = 88

x = 11

3(11) - 8 = 33 - 8 = 25 for <A

5(11) + 10 = 55 + 10 = 65 for<B

What is 1/5 miles in units to 1/65 hours

Answers

Answer:

13/65 miles in 1/65 of an hour, I believe is the answer you're looking for. Hope this helped!

Step-by-step explanation:

Write the following set using roster notation. Do not include repeats.
A is the set of odd integers between 4 and 12.

Answers

Answer:

The Set of Natural Numbers: {1, 2, 3, 4, 5, . . .}

The Set of Whole Numbers: {0, 1, 2, 3, 4, 5, . . .}

The Set of Integers: {. . . , −5, −4, −3, −2, −1, 0, 1, 2, 3, 4, 5, . . .}

Step-by-step explanation:

the sum of a number and 6 is 3

Answers

Answer:

x + 6 = 3

-3 + 6 = 3

Step-by-step explanation:

x + 6 = 3

x + 6 - 6 = 3 - 6

x = -3

Seven more than three times a number is 25. What is the number?

Answers

The answer is 6

Explanation
3 x 6 is 18
18 +7 is 25

What number belongs in the box to make the equation true? 2 7 3 5​

Answers

It would be 5, because 2 +3 =5 and 7-2=5

Sally hopes to buy a 10 - kayak for $15,000. Describe all the numbers that when round to the nearest hundred are 15,000

Answers

Answer:

All the numbers ranging from 14,500 to 15444 would be rounded to 15000

Step-by-step explanation:

All the numbers ranging from 14,500 to 15444 would be rounded to 15000.

How this is possible.

1) all the numbers from 0-4 are not counted as the next number when rounded off.

2) all the numbers from 5-9 are counted as the next number when rounded off.

Example Suppose we have numbers from 15- 19.

These numbers can be rounded off to 20

But numbers from 10-14 would never be rounded off to 20 instead they are rounded as 10.

Same happens in the hundreds case.

If we have numbers from 450 - 499 they can be rounded as 500

And numbers from 400 - 444 cannot be rounded as 500  but the number 445 may be rounded as 450 . When we carry  5 from units place and replace it as 450 then again 450 can be rounded to 500.

Same happens with 15000  but when are rounding nearest hundred not ten.

Thus all the numbers ranging from 14,500 to 15444 would be rounded to 15000.

Find an expression for the number of colored border tiles needed for any N x N square center.

Answers

Answer:

8N - 4 border tiles.

Step-by-step explanation:

There are 4 border tiles in each corner and between  the tiles there are N-2 tiles along the 4 sides.

So the number of border tiles

= 4+ 4(N - 2)

= 4 + 4N - 8

= 8N - 4.

Other Questions
According to the video, who is the most common employer for Foresters? lumber companies the government paper companies conservation groups The movement of the progress bar may be uneven because questions can be worth more or less (including zero) depending on your answerIt is currently 25C in Greensboro, NC. Use the formula F = $C + 32, where C = Celsius degrees and F =Fahrenheit degrees, to convert 25C to Fahrenheit degrees.0 45.9FO 13FO 77F102.6 FSubmitPassSave and closeDon't knowanswerpe here to searchOBI8a15 In which of the following biomes is annual precipitation lowest?a. tundrab. taigac. grasslandd. temperate deciduous foreste. tropical rain forest Find the two missing numbers shown with an "*" in the equation. 5 3/* * 1/2 = 19^^both mixed fraction^^ how to download gta5 Which part of the compound light microscope is used to change magnification?Part APart CPart EPart F WILL GIVE BRAINLIEST!!! A timeline would be MOST useful for understanding which of the following? Standards: 8.1.9.B Group of answer choices The events of WWII The movement of people across the Atlantic in 1607 The moon landing Lindberghs first solo flight Luke is going to school to get a degree in Veterinary Medicine. He wants to work in an animal hospital after he graduates.Which of the following job options would be most beneficial to Luke's career while he is going to school?a. Babysit after schoolb. Work at a fast food restaurantc. Volunteer at a social service agencyd. Work an entry level position at a veterinary clinic what algebraic expression represents GK? what are basic structural elements in a photograph what can you say about the average distance from the nuclease of an electron the 2's orbital as compared with a 3s orbital HOW DID PEOPLE SURVIVE IN THE YEARS of Paleolithic &The Metal Ages What is the cause of overfishing in developing countries Ron Landscaping's income statement reports net income of $73,300, which includes deductions for interest expense of $10,500 and income taxes of $32,900. Its times interest earned is: Which fraction is equivalent to a repeating decimal? A)3/11 B)3/4 C) 3/8 D) 3/5 Simplify :............ what type of elements are bonding to create an average star? what new elements are they creating? can someone help me with this question Fill in the correct preposition.1. You must provide ____ your children _____ the evil day. How many cells make up single gelled organisms?